2016 AIME II Problems/Problem 15

Revision as of 21:02, 17 March 2016 by Shaddoll (talk | contribs) (Solution)
(diff) ← Older revision | Latest revision (diff) | Newer revision → (diff)

For $1 \leq i \leq 215$ let $a_i = \dfrac{1}{2^{i}}$ and $a_216 = \dfrac{1}{2^{215}}$. Let $x_1, x_2, ..., x_215$ be positive real numbers such that $\sum_{i=1}^{215} x_i=1$ and $\sum_{i \leq i < j \leq 216 x_ix_j = \dfrac{107}{215} + \sum_{i=1}^{216} \dfrac{a_i x_i^{2}}{2(1-a_i)}. The maximum possible value of$ (Error compiling LaTeX. Unknown error_msg)x_2=\dfrac{m}{n}$, where$m$and$n$are relatively prime positive integers. Find$m+n$.

==Solution== Replace$ (Error compiling LaTeX. Unknown error_msg)\sum x_ix_j$with$\frac12\left(\left(\sum x_i\right)^2-\sum x_i^2\right)$and the second equation becomes$\sum\frac{x_i^2}{1-a_i}=\frac{1}{215}$. Conveniently,$\sum 1-a_i=215$so we get$\left(\sum 1-a_i\right)\left(\sum\frac{x_i^2}{1-a_i}\right)=1=\left(\sum x_i\right)^2$. This is the equality case of Cauchy so$x_i=c(1-a_i)$for some constant$c$. Using$\sum x_i=1$, we find$c=\frac{1}{215}$and thus$x_2=\frac{3}{860}$.